No video

Polar coordinate limit

  Рет қаралды 57,060

Dr Peyam

Dr Peyam

Күн бұрын

In this video I use polar coordinates to calculate a limit in several variables. This is usually very hard to do, but polar coordinates simplify our work tremendously.

Пікірлер: 108
@marcioamaral7511
@marcioamaral7511 6 жыл бұрын
More multivariable calc please, these videos are for sure great!
@plaustrarius
@plaustrarius 6 жыл бұрын
@0:57 I think you mean y=rsin(theta) on the whiteboard but you change it in the next line and when you are speaking so everything works out haha @3:57 when you multiple each term by r^2, you can make a tighter restriction on the lower bound because it was previously zero it's still zero so you don't even have to evaluate a limit for the lower bound.
@Aaron-lp3zt
@Aaron-lp3zt 2 жыл бұрын
I appreciate how happy you are doing this limit. I was stressed about my exam, but after your video it clicked.
@plaustrarius
@plaustrarius 6 жыл бұрын
The squeeze theorem is so important!!! I wish I understood this concept earlier, it somehow tangentially helped me understand L'Hospital's Rule, along with looking at the series expansions. Thank you for the upload!
@GabrielPohl
@GabrielPohl 5 жыл бұрын
I'm definetly gonna use this in my test, my teacher didn't see that coming! hahahahahahah
@Smoothcurveup52
@Smoothcurveup52 8 ай бұрын
Wonderful explaination sir The way of teaching is really awesome sir Full respect from India 🙏
@TheDiederikdehaan
@TheDiederikdehaan 5 жыл бұрын
Is the squeeze theorem necessary in this case though? Don't cos & sin have positive powers and therefore always larger than zero? I'm not sure - just observing.... Thank you Dr Peyam - love your videos!
@moskthinks9801
@moskthinks9801 6 жыл бұрын
The answer for the limit is the average/current amount of dislikes you have generally/on this vid.
@drpeyam
@drpeyam 6 жыл бұрын
❤️
@tolgakabacasamala
@tolgakabacasamala 3 жыл бұрын
Could you please consider on the limit of the function (y*x^3)/(y^2+x^6) at (0,0). I think polar coordinate approach does not work in this case. After polar coordinate switch, I calculate the limit as 0. On the other hand, as aproaching to (0,0) along the path y=x^3, the limit is 1/2. Is there anything wrong that I calculated? My reasoning is "polar coordinate is just representing the linear aproach to (0,0), because we are keeping theta as constant and r is approaching to 0. Then, if limit does not related to theta, this means all linear paths to (0,0) give same limit. But this approach can not represent curvilinear path approaches." At this point you may ask "why don't we use y=mx switch, instead of polar coordinates" y=mx represent all linear paths except x=0. By this point of view, I am not agree with your spiral approach explaining as well. What do you think about my reasoning?
@drpeyam
@drpeyam 3 жыл бұрын
No you did it correctly. Since you find 2 different limits, the limit does not exist
@rodjaknenad6984
@rodjaknenad6984 2 жыл бұрын
@@drpeyam But in the video your statement at 4:08 implies that this method tests all paths, but it apparently doesnt. If this does not test a curved path such as the one that the commenter here gave, how can we test all of the paths?
@xbz24
@xbz24 2 жыл бұрын
nice vid dr, just what i needed, huge greetings from chili
@michaelempeigne3519
@michaelempeigne3519 5 жыл бұрын
correction to your parameterization of x and y at beginning : x = r cos theta and y = r sin theta
@real0067
@real0067 11 ай бұрын
Sei il re di tutti i re!❤
@katiegalgano3351
@katiegalgano3351 10 ай бұрын
Awesome video!
@irok1
@irok1 Жыл бұрын
Thanks for watching at the beginning of the video? That's great, almost as great as the video
@amandaliu7409
@amandaliu7409 4 күн бұрын
very good
@armandoski-g
@armandoski-g 3 жыл бұрын
Hey Dr. Peyam, couldn't we use a similar argument to say that if theta is close to k*pi , rsin(theta) tends to zero and so y tending to zero is equivalent to theta tending to kpi? Shouldn't we divide the limits into the two cases; when r tends to zero and also the case theta tends to kpi (in the y case, the x case would be theta tending to kpi/2)?
@mathadventuress
@mathadventuress 4 жыл бұрын
Dr peyam in my class my professor had us evaluate it like a substitution Like we do y=MX and y=kx² If the both go to the same value, then the limit exists otherwise it doesn't So...can we do this same thing that way as well? And factor and cancel out any terms?
@BananaHead223
@BananaHead223 3 жыл бұрын
This helped me out a lot! Thank you!
@leonardromano1491
@leonardromano1491 6 жыл бұрын
I believe it is also possible to let x(n)=x0+a/n and y(n)=y0+b/n for arbitrary a and b and then let n--> infinity. If the result depends on your choice of a and b then the function is not continuous in (x0,y0). This works because you can have arbitrary paths depending on how you choose a and b.
@AQWorldsOmmiiT
@AQWorldsOmmiiT 2 жыл бұрын
Hey Dr. Peyam! At 3:14, there is a mistake in multiplying the inequalities by "r".
@tofu8676
@tofu8676 6 жыл бұрын
thaha i struggled so hard with the idea that you have to prove such limits without assumptions on the path you take in calc class ^^
@derekeby8654
@derekeby8654 2 жыл бұрын
You messed that up quite a bit to get the correct result :)
@elliewolcott6254
@elliewolcott6254 2 жыл бұрын
Thank you so much!!
@wesenfikre4821
@wesenfikre4821 5 жыл бұрын
It's really helpful thanks Dr 😊👍
@ompandey6345
@ompandey6345 3 жыл бұрын
Nicely teach i am maths teacher from India
@fazelbanoei6212
@fazelbanoei6212 3 жыл бұрын
haha thanks , you give wholesome vibes
@vasundhrabisht8941
@vasundhrabisht8941 3 жыл бұрын
Hi I tried to evaluate a limit using this. The function is x²y / (x^4 + y^2). And using this my answer came out to be zero but this limit doesn't exist. How do we know where to apply polar coordinates to evaluate limits and where not?
@OtherTheDave
@OtherTheDave 6 жыл бұрын
I used to just say that “y = mx” (b was 0, since the lines won’t intersect there if there’s an offset), and then see if the limit depends on m.
@dalek1099
@dalek1099 6 жыл бұрын
I don't think that works as you must consider all the possible ways x and y could go to zero not just straight lines. Peyam's proof works as it doesn't even matter whether theta varies on each path due to the squeeze theorem.
@OtherTheDave
@OtherTheDave 6 жыл бұрын
dalek1099 Does that matter? It always got the right answer in school, but that might’ve been luck.
@dalekonezeroninenine4588
@dalekonezeroninenine4588 6 жыл бұрын
Consider the limit as x and y go to 0 of 2x^2y/(x^4+y^2) if you let y=mx then this expression becomes 2mx^3/(x^4+m^2x^2)>2mx^3/(m^2x^2)=2x/m which has limit of zero. However, if you let y=x^2 then this expression becomes 2x^4/(2x^4)=1. You would be alright if you said y=m(x)x and then showed the limit was zero like theta in Peyam's video has to be a function of x and y.
@TechNewsAndReview101
@TechNewsAndReview101 4 жыл бұрын
Thank you SO MUCH!
@someone229
@someone229 6 жыл бұрын
A new video from Dr π times m! like before I watch 🙈
@marcioamaral7511
@marcioamaral7511 6 жыл бұрын
My professor uses rho as radius and phi as the angle for polar coordinates he likes weird notation
@PackSciences
@PackSciences 6 жыл бұрын
It's not weird, rho as radius is a valid international notation. Phi as an angle is, however, not very common, as it can be misinterpreted as spherical coordinates. Also I like weird notations too, it widens your culture of mathematics.
@marcioamaral7511
@marcioamaral7511 6 жыл бұрын
I didn't said that it wasn't valid though Saying weird was just an expression, I like the notation a lot actually We are at the current moment talking about multiple integrals and for the Jacobian he used Lagrange's notation for derivatives trying to keep it minimalistic He's from Vietnam
@tofu8676
@tofu8676 6 жыл бұрын
at our university we always write r and phi.
@PackSciences
@PackSciences 6 жыл бұрын
Tofu which uni is it?
@tofu8676
@tofu8676 6 жыл бұрын
PackSciences JKU in Linz, Austria most of the time we use (r,phi) for radial and (r,phi,theta) for spherical
@ashokkumarsah9258
@ashokkumarsah9258 3 жыл бұрын
Thanks Sir🙏 it's really fruitful 🙏🙏
@ashokkumarsah9258
@ashokkumarsah9258 3 жыл бұрын
From India
@aneeshsrinivas9088
@aneeshsrinivas9088 2 жыл бұрын
do general changes of coordinates work for these limits?
@colehonecker6436
@colehonecker6436 3 жыл бұрын
you are a gem :)
@Luis-lw8fr
@Luis-lw8fr 10 ай бұрын
Thank you😄
@Arup497
@Arup497 5 жыл бұрын
I found the center of c onic sections by following methodology: Step-1:let f(x,y)=0 be the equation of the conic section Step-2:then I calculated the partial derivative of f(x,y) with respect to x and y respectively. Step-3:Then,l equated those partial derivatives to zero and got 2 simultaneous equations Step-4:At last l solve those equations for (x,y) which was later found to be the coodinates of the conic section. I want to know the actual truth. So,l need your Noble help
@user-jr9xc4fo3t
@user-jr9xc4fo3t Жыл бұрын
thanks
@shubhumchatterjee
@shubhumchatterjee 3 жыл бұрын
Sir, one question is that will y= r sin(theta)?
@shubhumchatterjee
@shubhumchatterjee 2 жыл бұрын
@@paladise thanks bro
@abduhalloqovabdusamad3849
@abduhalloqovabdusamad3849 3 жыл бұрын
Thanks
@sreerajnairg
@sreerajnairg 8 ай бұрын
It’s not a supreme method to find if a function has a limit or not. You can use this to prove limit does not exist, yeah, but not to prove that limit exists or not. There might be other curve through which we may get a different limiting value.
@fashionlover620
@fashionlover620 2 жыл бұрын
Please do video on advanced calculus delta epsilon limit proof for function of two variable limit
@danielkirilov8065
@danielkirilov8065 6 жыл бұрын
Nicely done! Squeeeze.
@harelkariv1477
@harelkariv1477 3 жыл бұрын
Since x and y are practically "dummy variable" and they both go to 0, can't we assume x=y?
@drpeyam
@drpeyam 3 жыл бұрын
No you have to show it using all directions, x = y is the diagonal direction
@duncanw9901
@duncanw9901 6 жыл бұрын
can you do a video on Riemann's proof of the Fourier transform stuff? I tried to read through that on my own and it didn't work. :)
@drpeyam
@drpeyam 6 жыл бұрын
What do you mean?
@srpenguinbr
@srpenguinbr 6 жыл бұрын
As both variables approach the same value, I used the limit as x gos to 0 and switched y to x. I also got 0, is that a coincidence? Can I always do that?
@srpenguinbr
@srpenguinbr 2 жыл бұрын
@@paladise got it. Later I took Calc 2 and learned about the infinite number of paths I could take.
@_Jasleenkaur
@_Jasleenkaur 2 жыл бұрын
if limit exists or doesn't exist, we use polar coordinates. Why don't we check over other paths?
@drpeyam
@drpeyam 2 жыл бұрын
Polar coordinates is over all paths, it’s a different way of saying (x,y)
@federicopagano6590
@federicopagano6590 5 жыл бұрын
I knew this method and i loved it at first sight but i always was in doubt if this was legal to do now i can confirm that it is. But whats bothering me is if this is true (i dont doubt at all its perfectly done) why the hell in all courses keep on doing deltha epsilon to proove something wich it is much more easy to do in polar coordinates. If this "proove" that the limit is zero why is so popular the tradtitional hardwork? That question i really cannot answear the fact that we cannot deny is that in all courses do for this kind of limit the epsilon theta and thats the reason why i always was affraid to do this limit in polar coordinates i always think that there is maybe something hide that u cannot do. The statistics use a 99% epsilon delta in all courses the question is whyyyyy???? If this is perfectly done i feel like the statistics tell me dont ever think to do it this way but i dont kjow why , it seems to me perfectly done
@rizkyagungshahputra215
@rizkyagungshahputra215 6 жыл бұрын
please do more about multivariable limit
@danyaaaa31
@danyaaaa31 4 жыл бұрын
Amazing
@federicopagano6590
@federicopagano6590 4 жыл бұрын
It looks great but I always wondered why on earth considering y=mx at a fixed m approaching x=0 doesn't seem to be enough to conclude anything but at a fixed theta approaching r=0 works ,it must be an extra fact behind because in both cases we fix one variable and then approach the remaining one to zero but in one case works and in the other doesn't. And i think ..the reason is because in y=mx we cannot conclude anything but in polar we can ...lol we already know that,... the question is why
@flynnflanfck
@flynnflanfck 3 жыл бұрын
i think its because when you test y=mx, you only check along all straight lines. Therefore, you can't make any claims for other paths. However, using polar coordinates sets no constraints on the nature of your path
@TheRedfire21
@TheRedfire21 6 жыл бұрын
i want to note that it is not always true you can find a limit using polar coordinates, a classic example is f(x,y)=((x^2)y)/((x^4)+y^2) edt; typos
@mathadventuress
@mathadventuress 4 жыл бұрын
Wow neat
@meselmasal6602
@meselmasal6602 2 жыл бұрын
I think, to be able to use this property, we need the fact that f( rcos(theta)), rsin(theta))=F(r).G(theta)
@MrRyanroberson1
@MrRyanroberson1 6 жыл бұрын
Lets see if I can manage. The limit is (x4-y4+4y4)/(x2+y2) = (x2-y2)+4y4/(x2+y2), so we get 4*Lim(y4/(x2+y2)), and we can do the same backwards to find: (4x4-3x4+3y4)/(x2+y2), and we get 4*Lim(x4/(x2+y2)), to be expected. Maybe now we can go to 4*Lim(x2/(1+y2/x2)), and see that for all y2/x2 not approaching -1, that it approaches zero.
@drpeyam
@drpeyam 6 жыл бұрын
Watch the video...
@cycklist
@cycklist 6 жыл бұрын
Zero times r squared is not minus r squared :)
@OtherTheDave
@OtherTheDave 6 жыл бұрын
Portsmouth FC Unless r = 0
@PackSciences
@PackSciences 6 жыл бұрын
Portsmouth is refering to the fact that if he applies -r transform (before taking the limit operator), it should have been 0 as the bottom inequality, and not -r^2
@drpeyam
@drpeyam 6 жыл бұрын
Portsmouth FC Haha, I originally meant to say -1 < cos^4 < 1 and multiply it by r^2
@aboutmath2995
@aboutmath2995 6 жыл бұрын
thats just a greek Γ not r..
@anissawilkinson9647
@anissawilkinson9647 3 жыл бұрын
Thank you daddy
@davide467
@davide467 6 жыл бұрын
We also Need more ode
@drpeyam
@drpeyam 6 жыл бұрын
Ode are coming :)
@PackSciences
@PackSciences 6 жыл бұрын
What exactly did you smoke before this video? It's a short one indeed, but at first you messed up by writing r=cos(theta), but fortunately, you noticed it Then you wrote "y = r cos(theta)" while the line "x = r cos(theta)" was 2 cm above. I hoped you would have figured out it was sine when you replaced at 1:19 At 3:27, bottom boundary should be zero, if you apply *r^2 to all sides of the inequality.
@drpeyam
@drpeyam 6 жыл бұрын
:(
@PackSciences
@PackSciences 6 жыл бұрын
They are minor typos, but this video is dense in terms of error per time of video compared to your previous videos.
@marcioamaral7511
@marcioamaral7511 6 жыл бұрын
Try to be at least polite correcting other people We all make mistakes ,and more respect to Dr Peyam who takes his time to make these
@blackpenredpen
@blackpenredpen 6 жыл бұрын
Márcio Amaral agree!!!
@japotillor
@japotillor 6 жыл бұрын
Even the best mathematicians make mistakes...contrary to popular belief were not robots.
@nahombeyene7694
@nahombeyene7694 Жыл бұрын
Bro is excited
@federicopagano6590
@federicopagano6590 3 жыл бұрын
mmmm let's see if we take z=x.y^2/(x^2+y^4) at (0;0) and we approach with x=0 then it goes to 0 but if we take x=y^2 the limit goes to 1/2 so the limit does not exist. However if we plug x=r.cost and y=r.sin (t) we end up the whole thing going to zero!! So....the polar method fails here there must be some hidden conditions I must to check before using this I think it's not so easy to plug polar coordinates and make r goes to 0(which I confess I did the same for several years until I stumbled with this example today!!!)
@drpeyam
@drpeyam 3 жыл бұрын
I don’t think the polar method shows that it goes to 0, since you have r^3 / (r^2 + r^4) which doesn’t necessarily go to 0
@Sam-hc4sd
@Sam-hc4sd 3 жыл бұрын
Ok I thought he was some random Indian KZbinr!!
@drpeyam
@drpeyam 3 жыл бұрын
I’m Persian
@cezarvintea7240
@cezarvintea7240 2 жыл бұрын
The video is surely great but why does it look like you kidnapped some kids, tied them up and now you are explaining calculus to them?
@drpeyam
@drpeyam 2 жыл бұрын
😂😂😂
@exploringwithsd6253
@exploringwithsd6253 4 жыл бұрын
Lim (x^2-y^2)/(x^2+y^2),x,y tense to 0,0 =?????
@drpeyam
@drpeyam 4 жыл бұрын
Same thing
@MarcoMate87
@MarcoMate87 4 жыл бұрын
1:00 LOL
@EIYEI
@EIYEI 6 жыл бұрын
Y=r sin(theta)
@stefanj.8542
@stefanj.8542 3 жыл бұрын
Ba nebunule, ai grija ca acolo e sin la y, nu cos
@yousseftaha8464
@yousseftaha8464 9 ай бұрын
كسم الهند
Spherical Limit
4:02
Dr Peyam
Рет қаралды 11 М.
Limits of Multivariable Functions - Calculus 3
19:04
The Organic Chemistry Tutor
Рет қаралды 708 М.
❌Разве такое возможно? #story
01:00
Кэри Найс
Рет қаралды 1,8 МЛН
Get 10 Mega Boxes OR 60 Starr Drops!!
01:39
Brawl Stars
Рет қаралды 17 МЛН
Challenge matching picture with Alfredo Larin family! 😁
00:21
BigSchool
Рет қаралды 41 МЛН
BMS Parent Update_Aug 23, 2024
5:06
Boaz Middle Pirates
Рет қаралды 62
the DANGERS of POLAR COORDINATES
8:58
Michael Penn
Рет қаралды 28 М.
Limits and Continuity Using Polar Forms
12:28
JaberTime
Рет қаралды 5 М.
Rectangular coordinate vs polar coordinate (the formulas you need)
4:34
bprp calculus basics
Рет қаралды 2,8 М.
Limits of multivariable functions
11:35
Prime Newtons
Рет қаралды 25 М.
Area of Polar Curve r=1+2cos(theta)
7:28
blackpenredpen
Рет қаралды 155 М.
Multivariate Limits and the Squeeze Theorem - Examples
15:39
Mathemation
Рет қаралды 13 М.
Computing Multivariable Limits Algebraically
12:17
Dr. Trefor Bazett
Рет қаралды 62 М.
❌Разве такое возможно? #story
01:00
Кэри Найс
Рет қаралды 1,8 МЛН